Step 1: Understanding the fringe width formula. The fringe width in Young’s double-slit experiment is given by: \[ \beta = \frac{\lambda D}{d}, \] where: - \( \lambda \) is the wavelength of the light, - \( D \) is the distance between slits and screen, - \( d \) is the separation between the slits.
Step 2: Analyzing Assertion (A). Since \( \beta \propto \lambda \), red light (\(\lambda\) is larger) produces wider fringes than blue light (\(\lambda\) is smaller). Thus, Assertion (A) is incorrect because it states the opposite.
Step 3: Analyzing Reason (R). The fringe width is indeed proportional to the wavelength, which is a correct statement.
Since (A) is false but (R) is true, the correct choice is: \[ \boxed{\text{(2) (A) is false, but (R) is true.}} \]
Given below are two statements:
Statement I: In the oxalic acid vs KMnO$_4$ (in the presence of dil H$_2$SO$_4$) titration the solution needs to be heated initially to 60°C, but no heating is required in Ferrous ammonium sulphate (FAS) vs KMnO$_4$ titration (in the presence of dil H$_2$SO$_4$).
Statement II: In oxalic acid vs KMnO$_4$ titration, the initial formation of MnSO$_4$ takes place at high temperature, which then acts as catalyst for further reaction. In the case of FAS vs KMnO$_4$, heating oxidizes Fe$^{2+}$ into Fe$^{3+}$ by oxygen of air and error may be introduced in the experiment.
In the light of the above statements, choose the correct answer from the options given below: